返回列表 发帖

问一道逻辑JJ,此题为gwd的变体

In Gandania, where the government has a monopoly on tobacco sales,the incidence of smoking-related health problems has risen steadily for the last twenty years. The health secretary recently proposed a series of laws aimed at curtailing tobacco use in Gandania. Profits from tobacco sales,however, account for ten percent of Gandania’s annual revenues. Therefore,Gandania cannot afford to institute the proposed laws.

The argument above assumes that:
A.    All health care in Gandania is government-funded.
B.    Implementing the proposed laws is not likely to cause a significant increase in the amount of tobacco Gandania exports.
C.    The percentage of revenue Gandania receives from tobacco sales has remained steady in recent years.
D.    Profits from tobacco sales far surpass any other single source of revenue for the Gandanian government.
E.    No government official in Gandania has ever previously proposed laws aimed at curtailing tobacco use.
大家觉得这道题应该选什么?
收藏 分享

我也在疑惑这道题。我觉得肯定不选A

TOP

A是不是掉了一个单词  5个选项看起来都莫名其妙

TOP

A答案是完整的。我认为这道题选B。

TOP

我同意选B,其他选项更纠结

TOP

返回列表

站长推荐 关闭


美国top10 MBA VIP申请服务

自2003年开始提供 MBA 申请服务以来,保持着90% 以上的成功率,其中Top10 MBA服务成功率更是高达95%


查看